Transformación unitaria de Lorentz en espinor de Dirac cuantizado

Estoy atascado de nuevo en la página 59 de Peskin y Schroeder. En particular, no sé cómo obtienen la ecuación (3.110). Permítanme primero dar algunos antecedentes de la forma en que lo entiendo (pero podría estar completamente equivocado).

Un operador unitario tu ( Λ ) actúa sobre los estados de la siguiente manera:

| pag , s tu ( Λ ) | pag , s
y por lo tanto cualquier operador, como un campo de Dirac, se transforma como:
ψ ( X ) = tu ( Λ ) ψ ( X ) tu 1 ( Λ )
Ahora, de la ecuación (3.109):
tu ( Λ ) a pag s tu 1 ( Λ ) = mi Λ pag mi pag a Λ pag s
podemos encontrar la transformación de la solución de frecuencia positiva de ψ :
tu ( Λ ) ψ tu 1 ( Λ ) = tu ( Λ ) d 3 pag ( 2 π ) 3 1 2 mi pag s a pag s tu s ( pag ) mi i pag X tu 1 ( Λ )
tu ( Λ ) ψ tu 1 ( Λ ) = d 3 pag ( 2 π ) 3 1 2 mi pag s tu ( Λ ) a pag s tu 1 ( Λ ) tu ( Λ ) tu s ( pag ) tu 1 ( Λ ) mi i pag X
y usando la ecuación (3.109) esto se convierte en:
tu ( Λ ) ψ tu 1 ( Λ ) = d 3 pag ( 2 π ) 3 1 2 mi pag 2 mi Λ pag s a Λ pag s tu ( Λ ) tu s ( pag ) tu 1 ( Λ ) mi i pag X
y desde este punto no tengo idea de cómo llegar a la ecuación (3.110). Si alguien pudiera empujarme en la dirección correcta, se lo agradecería mucho. (Soy consciente de que la medida de integración es invariante de Lorentz).

Otra pregunta: ¿alguien tiene otras referencias/notas/libros donde discutan cómo se transforma el campo del operador de Dirac cuantificado ? Encuentro la explicación de P&S completamente confusa (como puede haber quedado claro a partir de las preguntas que he estado haciendo recientemente en este foro :)), pero no puedo encontrar ningún otro libro que trate este tema.

La ley de transformación A A = tu ( Λ ) A tu 1 ( Λ ) es para los operadores, pero tu s ( pag ) no es un operador, sino que es el coeficiente frente a los operadores de creación o aniquilación.
@JiaYiyang gracias por corregirme! Ahora he editado la publicación; esperemos que sea mejor ahora.
¿Has leído los autores dicen entre 109 y 110? En tu última ecuación, el tu tu tu 1 es solo tu desde tu es solo un numero Luego solo realiza un cambio de variables. pag ~ = Λ pag como ellos dicen.
Gracias por su respuesta. ¿Podrías explicarme por qué? tu s ( pag ) es solo un numero? Pensé que era un espinor de Dirac (vector de 4 componentes) que también depende de pag (y por lo tanto, pensaría que una transformación de Lorentz lo influiría). Trabajando bajo el supuesto de que tu s ( pag ) es solo un número, luego veo cómo puedo llegar a:
tu ( Λ ) ψ tu 1 ( Λ ) = d 3 pag ~ ( 2 π ) 3 1 2 mi pag ~ s a pag ~ s tu s ( Λ 1 pag ~ ) mi i pag ~ Λ X . Sin embargo, no entiendo por qué tu s ( Λ 1 pag ~ ) = Λ 1 / 2 1 tu s ( pag ~ ) . ¿Han derivado esto en alguna parte de su libro? ¿O puedo derivarlo observando transformaciones infinitesimales de Λ 1 / 2 1 ? ¿O hay algo más que me estoy perdiendo?
Estaba siendo esquemático. Para cada s y cada pag , el símbolo tu s ( pag ) es un número de la misma manera que los componentes mi i ( X ) del campo eléctrico son números para cada i y X . En cuanto al otro hecho, no estoy seguro de que lo demuestren en ninguna parte del libro, pero este es el tipo de pensamiento que realmente debería intentar probar por su cuenta. Creo que no debería necesitar apelar a transformaciones infinitesimales, pero no he tratado de probarlo yo mismo. Pruébelo, y le sugiero que haga otra pregunta si no puede resolverlo.
@joshphysics gracias, me das mucho que pensar. Intentaré encontrar una solución o tal vez la posponga hasta que empiece a leer a Weinberg. Me gustaría hacerle una pregunta más acerca de por qué [ tu ( Λ ) , tu s ( pag ) ] = 0 . Lo siento si esto es trivial, pero no entiendo por qué. tu s ( pag ) es un número para un dado pag y s (aunque entiendo por qué eso es cierto para mi i ( X ) ). Pensé que para un dado s y pag , tu s ( pag ) sigue siendo un vector de columna de cuatro componentes. La única razón por la que podría adivinar por qué viajan diariamente es porque tu s ( pag ) "vive" en un espacio diferente de donde el
matriz tu ( Λ ) actúa sobre ¿Es esto cierto, o estoy diciendo tonterías?
No, tienes toda la razón; Estaba siendo descuidado, al igual que Peskin y Shroeder con su notación. Realmente deberían escribir un índice de spinor en ψ para que la transformación lea tu ( Λ ) ψ α ( X ) tu 1 ( Λ ) = ( Λ 1 2 1 ) α β α ψ β ( Λ X ) . Lo que escribieron es solo una abreviatura de esa expresión completa. Esto significa que en la integral, tendrás ( tu s ( pag ) ) α que son números, y luego el cálculo se lleva a cabo como se desea.
¡Gracias hombre! Eso me aclara mucho y tiene sentido. Lo único que no entiendo en este momento es por qué tu s ( Λ 1 pag ~ ) = Λ 1 / 2 1 tu s ( pag ~ ) , pero trabajaré en esto por mí mismo y si no puedo resolverlo, lo haré como preguntas separadas. Salud
Ningún problema. ¡Salud! Por cierto, a menos que esté comentando el póster de una pregunta o respuesta, debe incluir @nombredeusuario. De lo contrario, no recibirá una notificación sobre el comentario.
@Hunter, ¿sabías por qué? tu s ( Λ 1 pag ) = Λ 1 2 tu s ( pag ) ? De hecho, lo hice como una nueva pregunta hace una hora. Luego encontré este mientras buscaba viejas preguntas esperando una respuesta. ¡Tan pronto como vi (3.110) en tu descripción supe de qué se trataría!
@ Flint72 jaja, no, nunca lo descubrí. Una vez que vi su pregunta, la marqué de inmediato, pero parece que Robin Ekman ha dado una buena respuesta (aunque necesito analizarla más a fondo). Es increíble cómo P&S a veces escribe fórmulas mágicamente sin ninguna explicación.

Respuestas (1)

Puedes encontrar la ley de transformación para tu s ( pag ) exigiendo que el campo espinoso se transforme como

ψ ( X ) ψ ( X ) = tu 1 ( Λ ) ψ ( X ) tu ( Λ ) = Λ 1 / 2 ψ ( X ) .

Ya sabe cómo se transforman los operadores de creación / aniquilación a partir de la condición de que los estados de 1 partícula se transformen correctamente y luego puede encontrar la ley de transformación correcta para tu s ( pag ) . Entonces, armado con esta ley de transformación, puedes hacer la transformación en la dirección opuesta (que es lo que hacen Peskin y Schroeder) y obtienes su resultado.

En particular, tenemos

tu 1 ( Λ ) ψ ( X ) tu ( Λ ) = d 3 pag ( 2 π ) 3 1 2 mi pag tu 1 ( Λ ) a pag s tu ( Λ ) tu s ( pag ) mi i pag . Λ X + términos similares

donde he ignorado la sumatoria y el otro operador ya que es análogo a esto.

Cambiar la variable ficticia pag a Λ pag obtenemos

tu 1 ( Λ ) ψ ( X ) tu ( Λ ) = d 3 Λ pag ( 2 π ) 3 1 2 mi Λ pag tu 1 ( Λ ) a Λ pag s tu ( Λ ) tu s ( Λ pag ) mi i pag . X

desde ( Λ pag ) ( Λ X ) = pag X

También tenemos tu 1 ( Λ ) a Λ pag s tu ( Λ ) = 2 mi pag 2 mi Λ pag a pag s dándonos

tu 1 ( Λ ) ψ ( X ) tu ( Λ ) = d 3 Λ pag ( 2 π ) 3 2 mi pag 2 mi Λ pag a pag s tu s ( Λ pag ) mi i pag . X

La medida es invariante de Lorentz, por lo que podemos reescribirla como

tu 1 ( Λ ) ψ ( X ) tu ( Λ ) = d 3 pag ( 2 π ) 3 1 2 mi pag a pag s tu s ( Λ pag ) mi i pag . X

Ahora exigimos que esto sea igual

Λ 1 / 2 ψ ( X ) = Λ 1 / 2 d 3 pag ( 2 π ) 3 1 2 mi pag a pag s tu s ( pag ) mi i pag . X

e inmediatamente vemos que debemos tener

tu s ( Λ pag ) = Λ 1 / 2 tu s ( pag ) .

Ahora puedes aplicar la transformación inversa, ψ ( X ) tu ( Λ ) ψ ( X ) tu 1 ( Λ ) para obtener el resultado de Peskin & Schroeder.

Lo siento pero tu respuesta es incorrecta. El error proviene de la transformación del operador de creación al que le falta la rotación de Wigner del pequeño grupo de partículas con giros. Es todo el asunto de los giros.
@TwoBs La rotación de Wigner no juega un papel cuando impulsamos en la dirección del giro o giramos sobre el eje de giro, que es el caso que están considerando Peskin & Schroeder.
ok, ya veo, te restringiste al caso trivial. No quedó claro en tu respuesta. ¿Podría editar su respuesta y agregar que está restringiendo a tal caso? Especialmente en la conclusión sobre la ley de transformación del espinor que no es la general. (O mejor aún, agregue el general). Después de sus ediciones, también podré eliminar los votos negativos.